AB
Round your answer to the nearest hundredth.
А
50°
6
12
B

ABRound Your Answer To The Nearest Hundredth.50612B

Answers

Answer 1

Answer:

h = 7.832

Step-by-step explanation:

This is a right angled triangle so, taking 50 as reference angle,

hypotenuse = ?

perpendicular = 6

The ratio for p and h is given by

Sin 50 = p/h

Sin 50 = 6 /h

h = 6 / Sin 50

h = 7.832


Related Questions

4. Solve the equation using the quadratic formula.
4x^2+3x-10 = 0
A.x= -2, x= 1.25
B.X= -2, x= 2
C.x= -1.25, x= 2
D.x= -1.25, x= 1.25​

Answers

Answer:

A. x = -2, x = 1.25

Step-by-step explanation:

Use the quadratic formula

x = [tex]\frac{-b+\sqrt{b^{2}-4ac } }{2a}[/tex]

Once in standard form, identify a, b, and c from the original equation and plug them into the quadratic formula.

4x² + 3x - 10 = 0

a = 4

b = 3

c = - 10

x = [tex]\frac{-3+\sqrt{3^{2}-4x4(-10) } }{2x4}[/tex]

Simplify

Evaluate the exponent

x = [tex]\frac{-3+\sqrt{9-4x4(-10)} }{2x4}[/tex]

Multiply the numbers

x = [tex]\frac{-3+\sqrt{9+160} }{2x4}[/tex]

Add the numbers

x = [tex]\frac{-3+\sqrt{169} }{2x4}[/tex]

Evaluate the square root

x = [tex]\frac{-3+13}{2x4}[/tex]

Multiply the numbers

x = [tex]\frac{-3+13}{8}[/tex]

Separate the equations

To solve for the unknown variable, separate into two equations: one with a plus and the other with a minus.

x = [tex]\frac{-3+13}{8}[/tex]

x = [tex]\frac{-3-13}{8}[/tex]

Solve

Rearrange and isolate the variable to find each solution

x = - 2

x = 1.25

Answer:

A. x = -2, x = 1.25

Step-by-step explanation:

Use the sum-product pattern

4x² + 3x - 10 = 0

4x² + 8x - 5x - 10 = 0

Common factor from the two pairs

(4x² + 8x) + (-5x - 10) = 0

4x (x + 2) - 5 (x + 2) = 0

Rewrite in factored form

4x (x + 2) - 5 (x + 2) = 0

(4x - 5)(x + 2) = 0

Create separate equations

(4x - 5)(x + 2) = 0

4x - 5 = 0

x + 2 = 0

Solve

Rearrange and isolate the variable to find each solution

x = 1.25

x = - 2

Let In be the number of n-digit quinary (0, 1, 2, 3, 4) sequences with (i) at least one 3 and (ii) the first 3 occurs before the first 0 (possibly no 0's). (Examples of 4-digit legal quinary sequences: 3120, 3000, 4123) (a) Show 91 = 1,92 = 8. = (b) Show a recurrence for an is qn = 39n-1 +5n-1 (91 = 1). = = 5" - 35 (c) A closed form for en is In = (n > 1). Finish the induction proof of this fact 2 (began below) by completing the induction step: 57 - 31 Base case (n = 1): LHS = q1 = 1. RHS 1 2 5k - 3k Induction Hypothesis: Assume true for n=k, i.e., Pk Induction Step: = II 2 N (d) Show how to derive this closed form, i.e., show how one can arrive at this closed form if they only knew the recurrence and the initial values.

Answers

Part a:Here, In be the number of n-digit quinary (0, 1, 2, 3, 4) sequences with(i) at least one 3 and (ii) the first 3 occurs before the first 0 (possibly no 0's).Since there is at least one 3 in the n-digit sequence, we start the sequence by selecting one of the 5 digits. There are five ways to do this.The next digits are chosen according to one of the three cases shown below:1) A string of (n-1) digits where no 0's are included in the string.2) A string of (n-1) digits where at least one 0 appears in the string before the first 3.3) A string of (n-1) digits where 3 appears before the first 0 in the string.The first string in case 1 can be formed in 4 different ways because no 0's can appear in the string and there are 4 digits to choose from (0, 1, 2, 4). There are 5 choices for the first digit and thus 5*4 quinary sequences of length n with at least one 3 and no 0's that start with the selected digit.The first string in case 2 can be formed in 5 different ways because the first 3 can appear in any position before the first 0. The remaining digits are chosen in n-2 positions because the first digit is already chosen (which is 3) and n-1 digits are left. There are 4 choices for each of the remaining n-2 positions because no 0's can appear in the string and there are 4 digits to choose from (0, 1, 2, 4). Thus, there are 5*5*4^(n-2) quinary sequences of length n with at least one 3 and at least one 0 that start with the selected digit.The first string in case 3 can be formed in n-1 different ways because the first 3 can appear in any position before the first 0. The remaining digits are chosen in n-2 positions because the first digit is already chosen (which is 3) and n-1 digits are left. There are 4 choices for each of the remaining n-2 positions because no 0's can appear in the string and there are 4 digits to choose from (0, 1, 2, 4). Thus, there are 5*(n-1)*4^(n-2) quinary sequences of length n with at least one 3 and at least one 0 that start with the selected digit.Therefore, the number of n-digit quinary sequences with (i) at least one 3 and (ii) the first 3 occurs before the first 0 (possibly no 0's) is the sum of the number of sequences in each of the three cases above, i.e.In = 5*4^(n-1) + 5*5*4^(n-2) + 5*(n-1)*4^(n-2)Part b:Recurrence: qn = 3q(n-1) + 4^(n-1) + 2.   q1 = 1. Let's see if qn = 39(n-1) + 5(n-1) satisfies this recurrence.q1 = 1 = 3(1-1) + 4^(1-1) + 2 = 0 + 1 + 2 = 3(0) + 5(1-1) + 1 = 0 + 0 + 1Thus, q1 = 1 satisfies the recurrence.qn+1 = 3qn + 4^n + 2 = 3(39n-1 + 5n-1) + 4^n + 2= 117(n-1) + 15(n-1) + 4^n + 2= 132(n-1) + 4^n + 3Using this formula, we can see that q2 = 91.Part c:Here, we need to finish the induction proof of this fact 2 that In = (n > 1).Induction Hypothesis: Assume true for n = k, i.e., Pk = Ik = 5*4^(k-1) + 5*5*4^(k-2) + 5*(k-1)*4^(k-2)Induction Step: To show that it is true for n = k+1, we need to show that the formula given above holds. The first digit can be any of the 5 digits (0, 1, 2, 3, 4) and the remaining digits can be selected in one of the three ways discussed in part (a).Case 1: n-1 digits with no 0'sThere are 4 choices for each of the n-1 digits, since 0 cannot be used. Therefore, there are 4^(n-1) such sequences with no 0's.Case 2: n-1 digits with at least one 0 before the first 3The first 3 can be in any of the n-1 positions, and the digits before it must be chosen from the set {0,1,2,4}. The remaining digits can be chosen in any of the 4 choices. Therefore, there are 5*(n-1)*4^(n-2) such sequences.Case 3: n-1 digits with 3 before 0We start by selecting one of the n-1 positions for the 3, then the remaining digits are chosen from the set {0,1,2,4}. There are (n-2) positions left for the remaining digits. There are 4 choices for each position, since 0 cannot be used. Therefore, there are 5*(n-1)*4^(n-2) such sequences.Thus, the total number of n-digit quinary sequences with at least one 3 and with the first 3 before the first 0 isIn = 5*4^(n-1) + 5*5*4^(n-2) + 5*(n-1)*4^(n-2) = qn+1 - qn = 132(n-1) + 4^n + 3 - (39(n-1) + 5(n-1)) = 93(n-1) + 4^n + 3which completes the induction proof.Part d:Since qn = 39n-1 + 5n-1, we haveqn+1 - qn = 132n - 39n - 5n = 88nTherefore, qn+1 = qn + 88nSubstituting qn = 39n-1 + 5n-1, we getqn+1 = qn + 88n = 39n-1 + 5n-1 + 88n = 39n + 5n + 88(n-1)Simplifying, we getqn+1 = 132(n-1) + 4^n + 3Therefore, en = In - In-1 = 93(n-1) + 4^n + 3 - 93(n-2) - 4^(n-1) - 3= 93n - 93(n-1) - 4^(n-1)= 93 - 4^(n-1)Thus, the closed form for en is en = 93 - 4^(n-1).

Learn more about n-digit here: brainly.com/question/17101915

#SPJ11

HELP PLEASE I WILL MARK BRAINST

Answers

Answer:

F=45

Step-by-step explanation:

G is 90 degrees and FC angles combined is 90 and divided is 45 each

HELP I NEED HELP ASAP
HELP I NEED HELP ASAP
HELP I NEED HELP ASAP
HELP I NEED HELP ASAP
HELP I NEED HELP ASAP
HELP I NEED HELP ASAP
HELP I NEED HELP ASAP
HELP I NEED HELP ASAP

Answers

Answer:

The answer is C

Step-by-step explanation:

The mean of math is 84, and the Mean of science is 85.

so, science is clearly one point higher

Find the lateral surface area.

Answers

Answer:

Step-by-step explanation:

5
7
9
4
In percents?

Answers

Answer:

Convert the decimal to a percentage by multiplying the decimal by  

100 .

500%

700%

900%

400%

Use a scientific calculator to find the Tangent of 180 degrees.

Answers

It’s 0. Hope that helps

which of the following functions are solutions of the differential equation y′′−9y′ 18y=0? a. y(x)=e6x b. y(x)=e−x c. y(x)=e3x d. y(x)=0 e. y(x)=6x f. y(x)=3x g. y(x)=ex

Answers

Only one of the following functions is a solution of the differential equation y′′−9y′+18y=0.

The second-order homogeneous linear differential equation is given as:y'' - 9y' + 18y = 0This differential equation is a linear homogeneous equation. We will have two roots of the characteristic equation: r1 = 3, r2 = 6So, the general solution to the differential equation is given as:y = c1e3x + c2e6xwhere c1 and c2 are arbitrary constants.a. y(x) = e6x is a solution because it is a part of the general solution of the differential equation.y(x) = e−x, y(x) = 0, y(x) = 6x, y(x) = 3x, y(x) = ex are not solutions because they don't satisfy the differential equation. Hence, the correct options are:a. y(x) = e6xTherefore, only one of the following functions is a solution of the differential equation y′′−9y′+18y=0.

Learn more about diffential equation here,

https://brainly.com/question/28099315

#SPJ11

Race) The longest racial grouping of respondents to the 2012 GSS was______, with ______%. The second-largest grouping was _____, with ______%.

Answers

The longest racial grouping of respondents to the 2012 GSS was non-Hispanic white, with 78.7%. The second-largest grouping was Black or African American, with 15.6%.

The General Social Survey (GSS) is a nationally representative survey of American adults that has been conducted annually since 1972. The GSS collects data on a wide range of topics, including race and ethnicity. In 2012, the GSS asked respondents to identify their race and ethnicity. The results showed that the largest racial grouping in the United States was non-Hispanic white, followed by Black or African American. in the 2012 GSS or any other related information, it is recommended to refer to the official documentation or reports from the General Social Survey (GSS).

To know more about ethnicity here : brainly.com/question/674712

#SPJ11








1. Find the following limits. 2x2 - 8 a) lim * 4x2 + 2 b) lim -0 c) lim 2x +5x+3 2x+3

Answers

The limit of the given expression as x approaches infinity is negative infinity. The limit of (4[tex]x^2[/tex] + 2) as x approaches negative infinity is positive infinity. The limit of (2x + 5x + 3) divided by (2x + 3) as x approaches negative infinity is 7/4.

In the first limit, as x approaches infinity, the dominant term in the expression is 2x^2. Since x^2 grows without bound as x becomes larger, the value of 2x^2 will also increase without bound, resulting in a limit of negative infinity.

In the second limit, as x approaches negative infinity, the dominant term in the expression is 4x^2. Similar to the first limit, since x^2 grows without bound as x becomes more negative, the value of 4x^2 will increase without bound, leading to a limit of positive infinity.

In the third limit, as x approaches negative infinity, both the numerator and denominator have the dominant term of 5x. Dividing the numerator and denominator by 5x, we get (2 + 5/x) divided by (2 + 3/x). As x approaches negative infinity, the terms with x in the denominator become negligible, resulting in the limit simplifying to 2/2, which equals 1. Therefore, the limit of (2x + 5x + 3)/(2x + 3) as x approaches negative infinity is 7/4.

Learn more about limit here:

https://brainly.com/question/12211820

#SPJ11

6) You are shooting the puck along the path
seen in the picture of the air hockey table.
Find the mZxº and the mzmº.

Answers

Answer:

m = 58°

x = 64°

Step-by-step explanation:

When a puck touches one side of the air hockey table,

All the angles formed are located at a point on a straight line.

Therefore, sum of angles formed is 180°.

m° + m° + 64° = 180°

2m + 64 = 180

2m = 180 - 64

2m = 116

m = 58°

m∠y = m = 58° [Alternate interior angles]

Similarly, on the other side of the table,

58° + y° + x° = 180°

58° + 58° + x° = 180°

x + 116° = 180°

x = 180 - 116

x = 64°

Write an expression that is equivalent to -4(3x – 7).
-4(3x – 7) =
– D
2 +
?

Answers

Another way to express it would be -12x+28

The function h defined by h(t)=(-4.9t + 29.4)(t+2) models the height in meters, of an object t seconds after it is launched. When will the object hit the ground?

Answers

Answer:

The time the object will hit the ground is 2 s.

Step-by-step explanation:

Given;

h(t) = (-4.9t + 29.4)(t + 2)

Open the bracket;

h(t) = -4.9t² + 29.4t  -9.8t + 58.8

h(t) = -4.9t²  + 19.6t + 58.8

When the  object hit the ground, the final velocity will be zero;

[tex]v = \frac{dh}{dt} = 0 \\\\\frac{dh}{dt} = -9.8t + 19.6 = 0\\\\9.8t = 19.6\\\\t = \frac{19.6}{9.8} \\\\t = 2 \ s[/tex]

Therefore, the time the object will hit the ground is 2 s.

If the coordinate of A is (0,-2) and the coordinate of B is (10,-6), then the midpoint of AB is (______).

Answers

Answer:

5, -4

Step-by-step explanation:

midpoint = x1 + x2/2 ,y1 + y2/2

= 0 + 10 /2 , -2 + -6/2

= 10/2 , -8/2

midpoint = 5, -4

A school recieved 20 boxes of different stickers.12 boxes contain floral stickers, 14 boxes were with 3-D stickers. There were 6 boxes of sparkly alphabet stickers. How many of 3-D floral stickers were received? Fill in the chart.

Answers

Answer:

14

Step-by-step explanation:

Answer:

14

Step-by-step explanation:

A circular flower bed is 16m in diameter and has a circular sidewalk around it that is 3 m wide. Find the area of the sidewalk in square meters. Use 3.14 for pi .

Answers

Answer:

[tex]178.98\ \text{m}^2[/tex]

Step-by-step explanation:

d = Diameter of flower bed = 16 m

Thickness of sidewalk = 3 m

r = Radius of flower bed = [tex]\dfrac{d}{2}=\dfrac{16}{2}=8\ \text{m}[/tex]

R = Radius of flower bed with sidewalk = [tex]8+3=11\ \text{m}[/tex]

The required area is given by

[tex]A=\pi(R^2-r^2)\\\Rightarrow A=3.14\times (11^2-8^2)\\\Rightarrow A=178.98\ \text{m}^2[/tex]

The radius of the sidewalk is [tex]178.98\ \text{m}^2[/tex].

A student selects three marbles of different color-One is red, the second blue and the third is green. He picks the marbles one at a time without replacement. What is the probability he selects a blue. followed by a red, and then green?

Answers

Here's an Overleaf PDF I created with an explanation for your question:

Please tell me if the transformation is a reflection or not!

Answers

Answer:

2, 3, & 5 are reflections, the others are rotations.

5x + 2 = x – 10 help

Answers

Answer:-3/4

Step-by-step explanation:

5x+2=x-1

5x-x=-1-2

4x=-3

X=-3/4

Which of the following indicate that the result from a simple linear regression model could be potentially misleading? a. The error terms follow a normal distribution b. The error terms exhibit homoscedasticity c. Then n th error term (e_n) can be predicted with e_n = 0.91 * e_n - 1 d. The dependent and the independent variable show a linear pattern

Answers

The correct answer is: c. The n-th error term (e_n) can be predicted with e_n = 0.91 * e_n - 1.

This statement indicates that there is a correlation or relationship between consecutive error terms, where the n-th error term can be predicted based on the previous error term. In a simple linear regression model, the error terms are assumed to be independent and have no correlation with each other.

However, if there is a correlation between the error terms, it violates the assumption of independence, which can lead to biased and unreliable regression results. Therefore, this condition suggests that the result from the simple linear regression model could be potentially misleading.

To learn more about linear regression, click here: brainly.com/question/30401933

#SPJ11

9-6x+x2 el dos va al cuadrado resuelvan lo porfA​

Answers

Answer:

6

Step-by-step explanation:

do 9-6 then

A business student conducts an OLS regression analysis in excel (with usual defaults) with demand for strawberries (in 1000 units) as the dependent and price (in dollars) as an independent variable. The OLS regression line is given by y= 9 − 3x. If the pvalue of the intercept coefficient is 0 and the pvalue of the slope coefficient is 2% & If the standard error of the intercept coefficient is 6 and the standard error of the slope coefficient is 1; the true slope will be ______ to/from the estimated slope and the true intercept will be ________ to/from the estimated intercept.

Group of answer choices

equal, equal

different, equal

equal, different

different, different

Answers

The true intercept and slope are both different from the estimated values based on statistical significance.

The p-value of the intercept coefficient and that of the slope coefficients are 0 and 0.02(2%) respectively. This means that they are statistically significant. Thus we can infer that the true intercept and slope is not equal to 0.

The standard error of the intercept coefficient is 6, which means that the true intercept is likely to be within 6 units of the estimated intercept. The standard error of the slope coefficient is 1, which means that the true slope is likely to be within 1 unit of the estimated slope.

Therefore, the true slope and intercept will be different from the estimated slope and intercept.

Learn more on regression analysis: https://brainly.com/question/28178214

#SPJ4

find the measure of each interior angle of a regular polygon with the following number of sides 4​

Answers

Answer:

90°

Step-by-step explanation:

Formula to find EACH interior angle:

[tex]\frac{(n-2) *180}{n}[/tex]

Given:

n (number of sides) = 4

Work:

[tex]\frac{(n-2) *180}{n} \\\\\frac{(4-2) *180}{4} \\\\\frac{(2) *180}{4} \\\\\frac{360}{4} \\\\90[/tex]

Susan found the equation of the best fit line for the data shown in the scatterplot. The slope of the line of best fit is

Answers

Answer:

Negative

Step-by-step explanation:

From the scatter plot displayed, we could clearly observe the direction of the trend line as it produces a negative slope. For high values of y, the values of x are low and similarly, high values of x have low y values. Therefore, this kind of relationship between the two variables is considered negative.

A group of students stood in a circle to play a game. The circle had a diameter of 22 meters. Which measurement is the closest to the circumference of the circle in meters?

Answers

Answer:

The closest measurement is 69 meters.

Step-by-step explanation:

We already know that 22 meters is the diameter and to find the circumference you need to multiply the diameter with π. We don't know π, so we are going with 3.14. When you multiply 22 and 3.14 together, you get 69.08 meters. You round the answer to the nearest meter, so it's 69 meters. So, the closest measurement is 69 meters.

HELP PLEASE AND ASAP!! look at screenshot (10 pts)

Answers

Answer:

C

Step-by-step explanation:

Add like terms to get

4.5a + 4b + 3.5c

Let A = {10,20,30}. Find one non-empty relation on set A such that all the given conditions are met and explain why it works: Not Reflexive, Not Transitive, Antisymmetric. (Find one relation on A that satisfies all three at the same time - don't create three different relations).

Answers

A non-empty relation on set A which satisfies all the condition is {(10, 20), (20, 30), (30, 10)}.

To find a non-empty relation on set A = {10, 20, 30} that satisfies the given conditions (not reflexive, not transitive, and antisymmetric), we can define the following relation:

R = {(10, 20), (20, 30), (30, 10)}

Explanation:

Not Reflexive: A relation R on set A is reflexive if for every element x in A, (x, x) is in R. In this case, (10, 10), (20, 20), and (30, 30) are not present in the relation R, which makes it not reflexive.

Not Transitive: A relation R on set A is transitive if for every (x, y) and (y, z) in R, (x, z) is also in R. In this case, we have (10, 20) and (20, 30) in R, but (10, 30) is not present. Therefore, the relation R is not transitive.

Antisymmetric: A relation R on set A is antisymmetric if for every (x, y) and (y, x) in R, where x ≠ y, then x and y are not the same element. In this case, we have (10, 20) and (20, 10) in R, but 10 is not equal to 20. Similarly, we have (20, 30) and (30, 20) in R, but 20 is not equal to 30. Therefore, the relation R is antisymmetric.

By defining the relation R as {(10, 20), (20, 30), (30, 10)}, we satisfy all three conditions simultaneously: not reflexive, not transitive, and antisymmetric.

To learn more about relation here:

https://brainly.com/question/32699461

#SPJ4

is it possible to have the ordered number pair 3;7 on the graph​

Answers

Answer:

Yes it is possible. Start at (0,0) which is the orgin and go right on the x-axis 3 spaces, and go up on the y-axis 7 spaces to get to the ordered pair of (3,7).

Step-by-step explanation:

2 8/100 turned into decimal

Answers

Answer:

2.08

Step-by-step explanation:

We know 2 is a whole number.

8/100 would be converted to the hundredths place.

0.000

     /\

      |

This is the hundredths place.

So the decimal is 2.08

---

hope it helps

Josh is starting a round of golf the first hole is 130 yards long he needs advice which club to use for his first shot yes kept careful rack for records and how close is for shots came to the olive from the same distance his records include data from his onto golf clubs a ledge and an eight iron over the past year

Answers

Answer:

hello your question is incomplete attached below is the complete question

a) Histogram shows positive skewness and the box plot depicts the fact that the median of the 3-iron club is bigger while the variance of the 5-wood club is more than that of 3-iron club when compared to their distance

b)The Median is a better choice because it is not affected by skewness or outliners

c) Josh should use Club 3 iron.   Reason : it has less variance

Step-by-step explanation:

A) Histogram and box plot is attached below

Histogram shows positive skewness and the box plot depicts the fact that the median of the 3-iron club is bigger while the variance of the 5-wood club is more than that of 3-iron club when compared to their distance

B) The Median is a better choice because it is not affected by skewness or outliners

Typical distance when Josh hits with a wedge  = 4 , distance when Josh hits with an 8 iron = 10.5

c) Josh should use Club 3 iron.   Reason : it has less variance

Other Questions
Find the volume. I hate these questions!! if y=4x^2 3 , what is the minimum value of the product \displaystyle xyxy ? Brainleist What Does Success Look Like After You've Been on the Job Six Months? Proving triangles similar What caused Martin Luther to seek change within the Catholic Church?please help meI will give you Ramen we compute the deviation of the second observation in the data set from the mean, and find that the result is a negative number. this tells us that What was Hideki Tojo's role in the Japanese government?AHe helped the military seize control and became prime minister.BHe overthrew Emperor Hirohito and was named the new emperor.CHe overthrew Emperor Hirohito and established a democratic government.DHe helped the military seize control and organized free and fair elections. Explain the economic impact of Italys location and trade on Renaissance art and culture. please help me with this problem i am stuck like chuck Pls helpThis partial circle has a radius of 11 inches.What is the area of this figure?Use 3.14 pi.Enter your answer as a decimal in the box. Round your answer to the nearest hundredth. in At a bookstore Mrs.McIntyre buys three books, one for each of her children. Each book cost the same amount. She also buys $1.50 magazine for herself. Her purchases have a total price of $13.47. The equation 3x+1.50=13.47 models this situation. Solve this equation for x. Explain what this value of x means in terms of the bookstore purchases. find the value of k if the rootsof 3x-5x+k=0, differ by 2 assume that in blackjack, an ace is always worth 11, all face cards (jack, queen, king) are worth 10, and all number cards are worth the number they show. given a shuffled deck of cards: Which type of testing is used to test how well the system will perform with a workload?Integration testingperformance testingunit testingacceptance testing Given that the point P(6.-7) lies on the line 6x + ky = -20, find k Need Help? Let p be the function defined by g(x) = -x2 + Bx. Find g(a + h), g(-a), g(sqrt a) a + g(a), and 1/g(a) 241.78 divided by (-3.85) The phrase, beating the dead horse idiom or metaphor? Or is it considered both? Write a Supreme Court opinion that describes your decision for United States v fields. Write three to four paragraphs. Find a equivalent for 24/100 The purpose of this exercise is to compare the variability of xy and with the variability of (x4 x2). Complete parts a through e below. --- a Suppose the first sample is selected from a population with mean 14 = 135 and variance of = 576. Within what range should the sample mean vary about 95% of the time in repeated samples of 64 measurements from this distribution? That is, construct an interval extending 2 standard deviations of Xy on each side of The interval is (10) (Simplify your answers. Type integers or decimals. Do not round.) Which one of following industries falls under provincial human rights jurisdiction? Entertainment industry O Transportation industry O Communication industry O Banking industry